[Math] Spectrum of the Resolvent of a Self-Adjoint Operator

functional-analysisoperator-theory

Let $\mathcal{H}$ be a Hilbert space, and $A$ a self-adjoint operator with domain $D_{A} \subseteq \mathcal{H}$. Assume that $\lambda_0 \in \rho(A)$, where $\rho(A)$ is the resolvent set of $A$. For any $z \in \rho(A)$, let $R_{A}(z)=(A – z I)^{-1}$ be the resolvent of $A$.

Choose $\lambda \neq \lambda_0$. Then it is well known that $\lambda \in \rho(A)$ if and only if $(\lambda – \lambda_0)^{-1} \in \rho(R_{A}(\lambda_0))$ (see e.g. Schmudgen, Unbounded Self-adjoint operators on Hilbert Space, Proposition 2.10). So we have (note that by the spectral theorem $\sigma(A)$ is nonempty):
\begin{equation}
\sigma(R_{A}(\lambda_0)) \backslash \{0\} = \left \{ \frac{1}{\mu – \lambda_0} : \mu \in \sigma(A) \right \}.
\end{equation}
If $A$ is a bounded operator on $\mathcal{H}$, then $0 \in \rho(R_{A}(\lambda_0))$, so that in this case, being $\sigma(A)$ closed, we have
\begin{equation}
\sigma(R_{A}(\lambda_0)) = \left \{ \frac{1}{\mu – \lambda_0} : \mu \in \sigma(A) \right \} = \text{closure} \left \{ \frac{1}{\mu – \lambda_0} : \mu \in \sigma(A) \right \}.
\end{equation}
Now suppose that $A$ is unbounded. In this case $0 \in \sigma(R_{A}(\lambda_0))$. If we could prove that $0$ is not an isolated point of $\sigma(R_{A}(\lambda_0))$ (which is the same to say that $\sigma(A)$ is not bounded), we could conclude also in this case that
\begin{equation}
\sigma(R_{A}(\lambda_0)) = \text{closure} \left \{ \frac{1}{\mu – \lambda_0} : \mu \in \sigma(A) \right \}.
\end{equation}
So my question is the following: if $A$ is unbounded, can $\sigma(A)$ be bounded?

PS This question arouse from the answer given by TrialAndError in this post
Norm of the Resolvent

Best Answer

Take $r > \max \sigma(A)$. Then $R_A(r)$ is self-adjoint and bounded. If $0$ is not in its spectrum, then $A = (R_A(r)^{-1}+rI$ is bounded. If $0$ is in its spectrum, it is an isolated point of the spectrum and therefore must be an eigenvalue: $R_A v = 0$ for some $v \in \mathcal H$. But that is impossible since $R_A(r) = (A-rI)^{-1}$, i.e. $R_A(r) v = u$ where $u \in D_A$ and $(A-rI) u = v$.

Related Question